Solve the equation 3x - 4y = 16 for x.16 4OA. X-B. x1643C. X= 4y + 16O D. x= 3(16+47)

Answers

Answer 1

To solve for x, first, we add 4y to the equation:

[tex]\begin{gathered} 3x-4y+4y=16+4y, \\ 3x=16+4y\text{.} \end{gathered}[/tex]

Now, we divide by 3:

[tex]\begin{gathered} \frac{3x}{3}=\frac{16+4y}{3}, \\ x=\frac{16+4y}{3}\text{.} \end{gathered}[/tex]

Answer:

[tex]x=\frac{16+4y}{3}\text{.}[/tex]


Related Questions

The next model of a sports car will cost 3.4% more than the current model. The current model costs $36,000. How much will the price increase in dollars? What will be the price of the next model?

Answers

ANSWER

[tex]\begin{gathered} 1224 \\ 37224 \end{gathered}[/tex]

EXPLANATION

Given;

Current model costs $36000

$36000 is 100% of current price.

Next model will be 100% plus 3.4%;

[tex]\begin{gathered} 100+3.4=103.4 \\ =\frac{103.4}{100} \\ =1.034 \end{gathered}[/tex]

t

[tex]\begin{gathered} 1.034\times36000 \\ =37224 \end{gathered}[/tex]

Therefore, the increase in price;

[tex]\begin{gathered} 37224-36000 \\ =1224 \end{gathered}[/tex]

Hence, the price increase in dollars is $1224 while the price of the next model is $37224

Ashley‘s Internet service is terribly unreliable in fact on any given day there’s a 60% chance that her Internet‘s connection will be lost at some point that day what is the probability that her Internet service is not broken for seven days in a row inner a fraction or round your answer to four decimal places if necessary.

Answers

Let the event that her internet will be broken be A

The event that her internet will not be broken be B

Therefore:

[tex]\begin{gathered} P(A)=60\%=0.60 \\ P(B)=1-0.60=0.4 \end{gathered}[/tex]

Thus, the probability that her internet is not broken for 7 days in a row:

[tex]P(B\text{ for 7 days\rparen=P\lparen B\rparen}\times\text{P\lparen B\rparen}\times\text{P\lparen B\rparen}\times\text{P\lparen B\rparen}\times\text{P\lparen B\rparen}\times\text{P\lparen B\rparen}\times\text{P\lparen B\rparen}[/tex]

Substitute the value:

[tex]P(B\text{ for 7 days\rparen=0.4}\times\text{0.4}\times\text{0.4}\times\text{0.4}\times\text{0.4}\times\text{0.4}\times\text{0.4=0.001634}[/tex]

Round to four decimal places is 0.0016

Answer: 0.0016

determine the number of real solutions for the following quadratic equation using the discriminate

Answers

Given equation:

[tex]y=x^2-3x-4[/tex][tex]a=1,b=-3,c=-4[/tex]

Discriminant:

[tex]\begin{gathered} b^2-4ac \\ (-3)^2-4(1)(-4) \\ =9+16 \\ =25 \end{gathered}[/tex]

Number of real solutions:

Since the discriminant is > 0 (that is ,it is a positive value)

The Lyon Restaurant Survey provides food, decor, and service ratings for some of the top restaurants across the United States. For 186 restaurants located in Boston, the average price of a dinner, including one drink and tip, was 48.60 Dollars. You are leaving on a business trip to Boston and will eat dinner 23 of these restaurants, randomly selected. Your company will reimburse you for a maximum of 50 dollars per dinner. Business associates familiar with these restaurants have told you that the meal cost at one-third of these restaurants will exceed 50 dollars.


a. What is the probability that none of the meals will exceed the cost covered by your company?
b. What is the probability that at most 12 of the meals will exceed the cost covered by your company? What is the probability that between 4 and 8 of the meals will exceed the cost covered by your company?
c. Calculate the expected number of restaurants that will exceed the cost covered by your company.
d. Calculate the probability of the first question by using the binomial distribution approximation. Therefore, in this case we will consider the possibility of repetition in the randomly selected restaurants. Define p=r/N as the success probability.N is the size of the population. r is the number of elements considered as successes in the population.
e. Calculate the probability of the second question by using the binomial disribution approximation.
f. Calculate the probability of the third question by using the binomial disribution approximation.
g. Calculate the expected number of the fourth question by using the binomial disribution aproximation

Answers

Using the binomial distribution, the probabilities are given as follows:

a. None: 0%.

b.

At most 12: 0.9814 = 98.14%.Between 4 and 8: 0.6249 = 62.49%.

c. The expected number of restaurants that will exceed the cost covered by your company is of 7.67.

Using the normal approximation, the probabilities are:

a. None: 0.0008 = 0.08%.

b.

At most 12: 98.38 = 98.38%.Between 4 and 8: 0.6121 = 61.21%.

The difference in these probabilities is due to the small sample size.

Binomial distribution

The formula for the probability of x successes is:

[tex]P(X = x) = C_{n,x}.p^{x}.(1-p)^{n-x}[/tex]

[tex]C_{n,x} = \frac{n!}{x!(n-x)!}[/tex]

In which the parameters are given by:

n is the number of trials of the experiment.p is the probability of a success on a single trial of the experiment.

Considering that you will eat dinner at 23 restaurants, and at around one-third of them the meal cost will exceed 50 dollars, the values of these parameters are given as follows:

n = 23, p = 1/3 = 0.3333.

The probability that none will exceed is P(X = 0), hence:

P(X = 0) = (1 - 0.3333)^23 = 0% (rounded).

The probability of at most 12 is:

P(X <= 12) = P(X = 0) + P(X = 1) + ... + P(X = 12).

Using a binomial distribution calculator with the given parameters, the probability is:

P(X <= 12) = 0.9814 = 98.14%.

The probability that between 4 and 8 dinners are paid is:

P(4 <= X <= 8) = P(X = 4) + P(X = 5) + P(X = 6) + P(X = 7) + P(X = 8)

Using a calculator, or the mass function P(X = x) and adding each probability, the desired probability is:

P(4 <= X <= 8) = 0.0493 + 0.0937 + 0.1405 + 0.1707 + 0.1707 = 0.6249 = 62.49%.

Normal approximation

The first step for the normal approximation is finding the mean and the standard deviation, as follows:

Mean = expected number: [tex]\mu = np = 23 \times 0.3333 = 7.67[/tex]Standard deviation: [tex]\sigma = \sqrt{np(1-p) = \sqrt{23 \times 0.3333 \times 0.6667} = 2.26[/tex]

The probability of none, using continuity correction, is P(X < 0.5), which is the p-value of Z when X = 0.5, hence:

(the p-value of Z is found using the z-score table).

[tex]Z = \frac{X - \mu}{\sigma}[/tex] (z-score formula)

Z = (0.5 - 7.67)/2.26

Z = -3.17

Z = -3.17 has a p-value of 0.0008.

Hence the probability is 0.0008 = 0.08%.

The probability of at most 12 is P(X <= 12.5), using continuity correction, which is the p-value of Z when X = 12.5, hence:

[tex]Z = \frac{X - \mu}{\sigma}[/tex]

Z = (12.5 - 7.67)/2.26

Z = 2.14

Z = 2.14 has a p-value of 0.9838.

Hence the probability is of 98.38 = 98.38%.

The probability of between 4 and 8 dinners being paid is P(3.5 <= X <= 8.5), which is the p-value of Z when X = 8.5 subtracted by the p-value of Z when X = 3.5, hence:

X = 8.5:

[tex]Z = \frac{X - \mu}{\sigma}[/tex]

Z = (8.5 - 7.67)/2.26

Z = 0.37

Z = 0.37 has a p-value of 0.6443.

X = 3.5:

[tex]Z = \frac{X - \mu}{\sigma}[/tex]

Z = (3.5 - 7.67)/2.26

Z = -1.85

Z = -1.85 has a p-value of 0.0322.

Hence the probability is:

0.6443 - 0.0322 = 0.6121 = 61.21%.

More can be learned about the binomial distribution at https://brainly.com/question/24756209

#SPJ1

look at the screenshots

Answers

Answer:

c for the first one and a for the second

Select the graph for the solution of the open sentence. Click until the correct graph appears. Ix| + 3 > 3

Answers

Given the sentence;

[tex]\mleft|x\mright|+3>3[/tex]

Subtracting 3 from both sides;

[tex]\begin{gathered} \mleft|x\mright|+3>3 \\ |x|+3-3>3-3 \\ \mleft|x\mright|>0 \end{gathered}[/tex]

Given the absolute value of x to be greater than zero, the range of value of x is;

[tex]\begin{gathered} x>0 \\ or \\ x<0 \end{gathered}[/tex]

Therefore, the correct graph of the solution is;

in a regular polygon a exterior angle 15° how many sides does the polygon have

Answers

Answer:

Explanations:

Sum of the exterior angles of a polygon = 360°

For a regular polygon, all the angles are equal:

mn = 360

where n = the number of sides

m = the size of an exterior angle

For m = 15°

15n = 360

n = 360/15

n = 24

Therefore, the polygon

Could you help me with this please is from apex

Answers

Answer:

Completing the table we have;

Explanation:

Given the table in the attached image, we want to complete the table;

[tex]\text{Interest is 1\% compounded monthly}[/tex]

For period 1;

simple interest;

[tex]i_1=Prt=100\times0.01\times1=\text{ \$1.00}[/tex]

Compound interest;

[tex]\begin{gathered} f_1=P(1+\frac{r}{n})^{nt}=100(1+\frac{1}{12})^{1(12)}=\text{ \$}101.00 \\ \text{ Interest = }101.00-100=\text{ \$1.00} \end{gathered}[/tex]

For period 2;

simple interest;

[tex]i_2=Prt=100\times0.01\times1=\text{ \$1.00}[/tex]

compound interest;

[tex]\begin{gathered} f_2=P(1+\frac{r}{n})^{nt} \\ P=f_1=101.00 \\ =101(1+\frac{1}{12})^{1(12)}=\text{ \$}102.01 \\ \text{Interest}=102.01-101=\text{ \$}1.01 \end{gathered}[/tex]

Total interest

simple interest;

[tex]i_t=i_1+i_2=1+1=\text{ \$2.00}[/tex]

Compound Interest;

[tex]\text{ Total interest}=1.00+1.01=\text{ \$2.01}[/tex]

Therefore, completing the table we have;

a is less than or equal to 10

Answers

The expression of the mathematical statement is a ≤ 10

How to represent the mathematical statement as an expression?

From the question, we have the following mathematical statement that can be used in our computation:

a is less than or equal to 10

The key statement less than or equal to in mathematics and algebra can be represented using the following symbol

less than or equal to ⇒ ≤

So, we have the following representation

a is less than or equal to 10 ⇒ a is ≤ 10

This implies that we rewrite the above expression as follows

So, we have

a is less than or equal to 10 ⇒ a ≤ 10

The above expression cannot be further simplified

So, we leave it like that

Hence, the mathematical statement when expressed as an expression is a ≤ 10

Read more about word problems at

https://brainly.com/question/29223808

#SPJ1

a. Rotate the letter W 180° around the origin. Then translate the image up 4 units. Draw the final image. What new letter did you form? b. Is the new letter congruent to the original letter? Explain.

Answers

ANSWER and EXPLANATION

We have letter W on the graph.

The cordinates of its vertices are:

(0, 4), (1, 0), (2, 2), (3, 0), (4, 4)

Now, on a cartesian plane, (x - y plane), we have 4 quadrants. The letter is on the first quadrant.

Because it rotates 180 degrees around the origin, it means that it mmoves by 2 quadrants:

So, it moves from quadrant 1 to quadrant 4.

The new cordinates become:

(0, -4), (-1, 0), (-2, -2), (-3, 0), (-4, -4)

Then it is translated 4 units up, so we add 4 units to each of the y values (Remember that cordinates are written as (x, y)):

(0, 0), (-1, 4), (-2, 2), (-3, 4), (-4, 0)

Now, plot those:

a) It forms the letter M.

b) For one shape to be congruent to another, it means that they have the same size. So, yes, the M is congruent to the W.

I need help if u need a pic of the graph I’ll take a picture of it

Answers

A.

Using the points (2,3) and (0,6) to find the slope (m), we have:

[tex]m=\frac{y2-y1}{x2-x1}=\frac{6-3}{0-2}=\frac{3}{-2}[/tex]

The slope is m= -3/2

B.

Using the points (-1, 7.5) and (1, 4.5) to find the slope (m), we have:

[tex]m=\frac{y2-y1}{x2-x1}=\frac{4.5-7.5}{1-(-1)}=\frac{-3}{1+1}=\frac{-3}{2}[/tex]

The slope is m= -3/2

C.

The slope is the same as we are finding the ratio of the vertical change to the horizontal change between two points. Since the function represents a linear equation the slope is going to be the same despite of the points you choose.

A washer and a dryer cost $765 combined. The washer costs $85 less than the dryer. What is the cost of the dryer?

Answers

The equation is formed and solved below

What is an equation?

Algebra is concerned with two types of equations: polynomial equations and the particular case of linear equations. Polynomial equations have the form P(x) = 0, where P is a polynomial, while linear equations have the form ax + b = 0, where a and b are parameters, when there is only one variable. To solve equations from either family, algorithmic or geometric approaches derived from linear algebra or mathematical analysis are used. Algebra also investigates Diophantine equations with integer coefficients and solutions. The approaches employed are unique and derive from number theory. In general, these equations are complex; one frequently searches just for the existence or lack of a solution, and, if they exist, the number of solutions.

Let the price of washer = $x

The cost of dryer = $x+85

The equation is formed as

x + x + 85 = 765

or, 2x = 765 - 85

or, x = 680/2 = 340

Price of dryer = $(340+85) = $425

To know more about equations, click on the link

https://brainly.com/question/27893282

#SPJ9

This problem is related to the linear equation and the required cost of the dryer is $425.

What is a linear equation?

If a variable's maximum power is always 1, an equation is said to be a linear equation. As a one-degree equation, it also goes by that name.

Let a washer costs be [tex]w[/tex] and a dryer costs be [tex]d[/tex].

Since the total cost of a washer and a dryer is $765, it follows:

[tex]w+d=765[/tex]                ... (1)

Further, it is given that the washer costs $85 less than the dryer, it means that:

[tex]w=d-85[/tex]                 ... (2)

Using the two linear equations (1) and (2), it follows:

[tex]d-85+d=765\\2d-85=765\\2d=765+85\\2d=850\\d=\frac{850}{2}=425[/tex]

Therefore, the cost of a dryer is $425.

To learn more about linear equations from the given link

https://brainly.com/question/26310043

#SPJ9

Does the formula represent a linear or nonlinear function? Explain

Answers

A linear function is an equation in which each term is either a constant or the product of a constant and the first power of a single variable. In other word, a linear function represents a straight line.

In our case, we have 2 variables: the volume (V) and the radius (r). However, the relationship is not linear because the radius is raised to the third power (not the first power). Therefore, the volume formula is a nonlinear function.

Determine the value of x Round results to an appropriate number of significant digits

Answers

Given

Find

The value of x.

Explanation

length of AB = 22 - 3 = 19

using the trignometric ratios , we have

[tex]\begin{gathered} \sin13\degree=\frac{BD}{AB} \\ \sin13\degree=\frac{\frac{x}{2}}{19} \\ \sin13\degree\times38=x \\ 8.548=x \end{gathered}[/tex]

Final Answer

Therefore , the length of x is 8.548

I need help graphing 3x+y=-1I already found the x intercept= -1/3

Answers

Here, we want to graph the line

To do this, we need to get the y-intercept and the x-intercept

The general equation form is;

[tex]y\text{ = mx + b}[/tex]

M is the slope while b is the y-intercept

Let us write the equation in the standard from;

[tex]y\text{ = -3x-1}[/tex]

The y-intercept is -1

So we have the point (0,-1)

To get the x-intercept, set y = 0

[tex]\begin{gathered} 0\text{ = -3x-1} \\ -3x\text{ = 1} \\ x\text{ = -}\frac{1}{3} \end{gathered}[/tex]

So, we have the x-intercept as (-1/3,0)

Now, if we join the two points, we have successfully graphed the line

The product of two integers is -24. The difference between the two integers is 14. The sum of two integers is 10. What are the two integers?

Answers

Answer:

12 & -2

Step-by-step explanation:

Brody received a $13.25 tip on a meal that cost $109. What percent of the meal costwas the tip?Round answer to the nearest whole percent.

Answers

Explanation

To find the percentage of the tip we will use the formula below.

[tex]\text{\%Tip}=\frac{\text{Tip(\$)}}{Cost\text{ of meal}}\times100[/tex][tex]\begin{gathered} \text{ \%Tip =}\frac{\text{13.25}}{109}\times100 \\ =13\text{\%} \end{gathered}[/tex]

Answer: 13%

Consider the functions below.Represent the interval where both functions are increasing on the number line provided.

Answers

The function f(x) is increasing for the intervals:

[tex]\begin{gathered} x\in(-\infty,-2\rbrack \\ x\in\lbrack2,\infty) \end{gathered}[/tex]

The midpoint of AB is M(7,-2). If the coordinates of A are (8,3), what are thecoordinates of B ?

Answers

The coordinates are ordered pairs with the x value listed first.

The change in x position is, 8-7=1

The change in y position is, 3-(-2)=5

Since the midpoint is halfway between A and B, the change will stay the same,

So, for B,

x is 7-1=6

y is -2-5=-7

The coordinnates of B is (6,-7)

0.0032% in fraction

Answers

Recall that the x% in fraction form is:

[tex]\frac{x}{100}\text{.}[/tex]

Therefore 0.0032% as a fraction is:

[tex]\frac{0.0032}{100}=\frac{\frac{32}{10000}}{100}\text{.}[/tex]

Simplifying the above result we get:

[tex]\frac{\frac{32}{10000}}{100}=\frac{32}{100\times10000}=\frac{1}{31250}\text{.}[/tex]

Answer:

[tex]\frac{1}{31250}[/tex]

Determine the domain and the range of the function.C. Determine where the function is increasing and where it is decreasing.

Answers

Given:

[tex]f(x)=2x^2-x+1[/tex][tex]a=2\text{ ; b= -1 ; c=1}[/tex]

Graph opes upwards.

Let the vertex be (h,k)

[tex]h=-\frac{b}{2a}[/tex][tex]h=-\frac{(-1)}{2(2)}[/tex][tex]h=\frac{1}{4}[/tex][tex]k=f(h)[/tex][tex]k=2(\frac{1}{4})^2-\frac{1}{4}+1[/tex][tex]k=2(\frac{1}{16})-\frac{1}{4}+1[/tex][tex]k=\frac{1}{8}-\frac{1}{4}+1[/tex][tex]k=\frac{1-2+8}{8}[/tex][tex]k=\frac{7}{8}[/tex][tex]\text{Vertex}=(\frac{1}{4},\frac{7}{8})[/tex]

Axis of symmetry is

[tex]x=\frac{1}{4}[/tex]

y- intercept

x=0,

y=1

There is no x intercept .

Domain:

[tex](-\infty,\infty)[/tex]

Range:

[tex]\lbrack\frac{1}{4},\infty)[/tex]

The function is increasing:

[tex](\frac{1}{4},\infty)[/tex]

The function is decreasing:

[tex](-\infty,\frac{1}{4})[/tex]

Use the graph to find the horizontal asymptote of the rational function

Answers

Horizontal Asymptote

Observing the graph with the red dashed line, the horizontal asymptote of the function is at y = 6

Vertical asymptote

If we draw a line the graph we have the following

This indicates that the vertical asymptote is at x = 2.

Answer this fraction based Question I will make you btainliest & provide you 50 points​

Answers

Answer:

i) 2/3, ii) 2/9,iii) 4/27,iv) Rs. 40000.

Step-by-step explanation:

i)

A person gives 1/3 of his wealth to his wife, then he is left with:

1 - 1/3 = 2/3 of the total amount

ii)

Then he gives 1/3 of the remainder to his son, the son gets:

2/3*1/3 = 2/9 of the total amount

iii)

The remaining portion is:

2/3 - 2/9 = 6/9 - 2/9 = 4/9 of the total amount

Each daughter gets 1/3 of it as there are three daughters:

4/9 * 1/3 = 4/27 of the total amount

iv)

If the total amount is x, the son gets 2/9x and a daughter gets 4/27x and the difference of the two is Rs 20000:

2/9x - 4/27x = 200006/27x - 4/27x = 200002/27x = 20000x = 20000*27/2x = 270000

This is the total amount.

The amount obtained by a daughter is:

4/27*270000 = 40000

x³ - 3x = 37

Help please :(

Answers

This problem has no solution in real numbers.There are two solutions for complex numbers.[tex]x_{1}=-1.8157-2.6261i[/tex][tex]x_{2}=-1.8157+2.6261i[/tex]

If you select one card at random from a standard deck of 52 cards, what is the probability of that card being a 5, 6 OR 7?

Answers

To solve this question we will use the following expression to compute the theoretical probability:

[tex]\frac{\text{favorable cases}}{total\text{ cases}}.[/tex]

1) We know that there are 4 fives, 4 sixes, and 4 sevens in a standard deck of 52 cards, then, the probability of selecting a 5, 6, or 7 is:

[tex]\frac{4+4+4}{52}\text{.}[/tex]

2) Simplifying the above expression we get:

[tex]\frac{12}{52}=\frac{3}{13}\text{.}[/tex]

Answer:

[tex]\frac{3}{13}\text{.}[/tex]

if x=10 units, then what is the volume of the cube

Answers

Knowing that the solid is a cube, you can use the following formula for calculate its volume:

[tex]V=s^3[/tex]

Where "s" is the length of any edge of the cube.

In this case, you can identify that:

[tex]s=x=10units[/tex]

Mrs walters had a bag full of candy she wanted to share with 18 students. If she had 335 pieces of candy how many pieces will each student get

Answers

Each student will get 18 pieces of candy. 18x18=324 or 335/18=18remainder,leftovers 11

Can you please answer this question for me. I don’t want full explanation I just want the answers

Answers

we have the fractions

1/4 and 3/4

Remember that

If the denominators are the same, then the fraction with the greater numerator is the greater fraction

3/4 > 1/4

use the number line

Divide number 1 into 4 parts

ILL GIVE BRAINLY AND 15 POINTS

Answers

Using functions, it is found that:

a. The costs for two years are as follows:

Option A: $4,600.Option B: $1,720.

b. The pros and cons are as follows:

Option A: pro is the lower fixed fees, con is the higher hourly fee.Option B: pro is the lower hourly fee, con is the higher fixed fees.

Option A function

The cost of $2,600 for the first year is obtained as follows:

$500 set up fee.$2,000 of the 100 hours at $20 per hour.$100 of the hosting fee.

For the second year, only the hourly cost will be paid, hence $2,000 will be added and the total cost is of:

$2,600 + $2,000 = $4,600.

The pros of Option A are the lower initial fees, hence for a lower number of hour, the cost is smaller, while the con is the high hourly price, meaning that for a high number of hours, option B is better.

Option B function

The cost of $2,600 for the first year is obtained as follows:

$1,000 set up fee.$30 a month of hosting fee.

For the second year, only the hosting fee is paid, meaning that $360 is added to the cost, thus:

$1,360 + $360 = $1,720.

The pros and cons are basically opposite of option A, higher basic fees with lower hourly/monthly fees, meaning that it is better over longer periods.

More can be learned about functions at https://brainly.com/question/24808124

#SPJ1

A spinner has the sections A through F. The spinner is spun and a 6-sided die is rolled. What is the probability that the outcome will be D and 5?1/361/181/121/6

Answers

To find the probability of having a D and %, we would use the concept of mutually exclusive events here

But probability is given as

[tex]P=\frac{\text{ number of favourable outcomes}}{\text{total number of }possible\text{ outcomes}}[/tex]

The probability of choosing a D is

A, B, C, D, E and F. This can be found as

[tex]P_a=\frac{1}{6}[/tex]

The probabilty of choosing a 5 out of 6 possible outcomes is

[tex]P_n=\frac{1}{6}[/tex]

The probability of having a D and 5 would be

[tex]\begin{gathered} P=P_a\times P_n \\ P=\frac{1}{6}\times\frac{1}{6} \\ P=\frac{1}{36} \end{gathered}[/tex]

From the calculations above, the answer to this question is 1/36

Other Questions
6. A profit function for a new business follows the functionP(x) = 1/3x^2 - 6x, where x represents the number of months.After how many months will the company begin to make aprofit?A. 2B. 9C. 12D. 18 how many ways are there for her to plan her schedule of menus for the 20 school days if there are no restrictions on the number of times she cooks a particular type of meal? consider a mutation that causes a decreased activity in a particular protein. a second mutation alters the function of another protein that participates in the same cellular pathway, thereby overcoming the defect in the first protein. this is an example of a(n) . Write an exponential function in the form y and (2,171). ab that goes through points (0,19) En un depsito haba 127 bolsas de harina cada una de 60 kg se sacaron ocho camiones de 12 bolsas cada uno cuantos kilogramos de harina quedaron en el depsito a line with a slope of 1/3 and containing the point (-4,7) A cheetah is running at a velocity of 8 m/s when it accelerptes at 1.5 m/s^2 for 6 seconds. If thecheetah started at a position of 20m what is the final Position of the cheetah? How do i dilate a scale factor by 2? The perimeter of a parallelogram is 76 meters. The width of the parallelogram is 2 meters less that its length. Find the length and the width of the parallelogram. if steak and potatoes are complements, when the price of steak goes up, the demand curve for potatoes: What is the polar form of the equation? What type of polar curve is this? which of the following internal record-keeping methods can a parent choose to account for a subsidiary acquired in a business combination? multiple choice initial value, equity, or partial equity. initial value, lower-of-cost-or-market-value, or equity. initial value, equity, or book value. initial value, lower-of-cost-or-market-value, or partial equity. initial value or book value. has preferred stock that pays a dividend of $1.25every year. if the discount rate on investments like this is 6.2%, what is the value of the preferred stock? PLSSS HELP IF YOU TURLY KNOW THISS algebra 2 question.. The scatter plot shows students scores for quiz 1 and quiz 2. a. What is the quiz 1 score for a student who earned a score of 13 on quiz 2? b. Did any student(s) earn the same score on both quiz 1 and quiz 2? Explain. c. The dotted line shows the line of best fit. Write its equation and then interpret the meaning of the slope and y-intercept. Does the y-intercept make sense in the context of the problem? The slope should be represented as a fraction or whole number just to let you know. Here is a picture attached of the graph. An air bubble has a volume of 0.510 L at 19 C. What is the volume, in liters, at 6 C , if n and P do not change? Which number sentence is true? A. |10| < 1.0 B. |0| = 0 C. |-47| < |-30| D. |-100| < |0| 6. There were a total of 139 points scored in the basketballgame. The Falcons scored (9x+22) and the Mavericks scored(16x - 8). What was the Falcons score for the game? The total fixed costs of producing a product is $55,000 and the variable cost is $190 per item. If the company believes they can sell 2,500 items at $245 each, what is thebreak-even point?800 items900 items960 items 1,000 itemsNone of these choices are correct.